primi p e q...

Numeri interi, razionali, divisibilità, equazioni diofantee, ...
Rispondi
Avatar utente
Reginald
Messaggi: 137
Iscritto il: 24 gen 2009, 15:52
Località: Trento

primi p e q...

Messaggio da Reginald »

trovare tutti i primi p e q tali che $ |p^m-q^n|=1 $ con m ed n maggiori di uno..

Non usate catalan, please.. :P
Veluca
Messaggi: 185
Iscritto il: 27 dic 2008, 01:08
Località: Chiavari (Genova)

Messaggio da Veluca »

$ p^m,q^n\equiv1 \vee 3 \pmod 4 $ se p,q>2. Infatti p e q sono dispari e quindi congrui a 1 o a 3 e le potenze di 3 sono 1 o 3 mod 4. In questo caso l'equazione è impossibile, in quanto 1-1, 1-3, 3-1, 3-3 non danno mai 1 in valore assoluto. quindi p=2 oppure q=2
caso 1) $ 2^m-q^n=1 \Rightarrow q^n=2^m-1=2^{m-1}+\dots+1 $ Sostituendo si ha $ 2^m=2^{m-1}+\dots+2^2+4 $. Si può quindi dividere per 4, supponendo m>2. e ottenere $ 2^{m-2}=2^{m-3}+\dots+4+2 $. Dividendo ancora per 2, se m>3, si ha un assurdo mod 2, quindi m=2 o m=3.
se m=2 si ha $ q^n=3 $, quindi n=1 -> contro le ipotesi. Allora m=3 -> $ q^n=7 $, quindi n=1 -> contro le ipotesi.
caso 2) $ 2^m-q^n=-1\Rightarrow q^n=3\cdot(2^{m-1}+\dots+1) $ se m è dispari. Quindi 3|q, ma q è primo, quindi q=3. Una coppia è quindi (2,3).
se m è pari, pongo m=2m':
$ 2^{2m'}-q^n=-1\Rightarrow $. Se anche n fosse pari, si avrebbe $ (2^{m'}-q^{n'})(2^{m'}+q^{n'})=-1 $, che è assurdo, in quanto è impossibile che j-k=-1 e j+k=1 o viceversa se $ j\ne0 $. Quindi n è dispari.
si ha quindi $ 2^m=(q-1)(q^{n-1}+\dots+1) $, quindi
$ \begin{cases} q=2^a+1\\ (2^a+1)^{n-1}+\dots+1=2^{m-a} \end{cases} $
analizzando la seconda modulo 2 si hanno 3 casi:
A) a=0 --> 1=0 (mod 2) assurdo (n,m>1 per ipotesi)
B) m=a --> n=1 (mod 2) e va bene
C) $ a\ne0 \wedge a\ne m $ -> n=0 (mod 2), assurdo perchè n è dispari
se a=m, allora $ (2^a+1)^{n-1}+\dots+1=1\Rightarrow n=1 $, contro le ipotesi
i casi 3 e 4 sono simmetrici, quindi le uniche soluzioni sono (2,3) e (3,2)
Avatar utente
Federiko
Messaggi: 226
Iscritto il: 15 mag 2008, 19:24
Località: Roma

Messaggio da Federiko »

Quando dividi $ 2^m=2^{m-1}+...+2^4+2^3+2^2+(2+1+1) $ per 4 ottieni $ 2^{m-2}=2^{m-3}+...+4+(\frac{8}{4}+2\frac{4}{4})=2^{m-3}+...+4+\textbf{4} $. In effetti poteva venirti il dubbio, perché hai dimostrato che $ \forall 2^m, m=2 \vee m=3 $ ...
CUCCIOLO
Veluca
Messaggi: 185
Iscritto il: 27 dic 2008, 01:08
Località: Chiavari (Genova)

Messaggio da Veluca »

:oops: vediamo di rimediare...
$ 2^m-q^n=1 $, quindi $ 2(2^{m-1}-1)=q^n-1=(q-1)(q^{n-1}+\dots+1) $. Quindi se n è pari e q è diverso da 2 si ha un assurdo mod 4. (se q=2 è ovviamente impossibile che due potenze di 2 maggiori di 2 abbiano come differenza 2)
se n è dispari si può scrivere $ 2^m=(q+1)(q^{n-1}-\dots+1) $
si ha quindi il sistema
$ \begin{cases} q=2^a-1\\ (2^a-1)^{n-1}-\dots+1=2^{n-a} \end{cases} $
quindi $ (-1)^{n-1}-\dots+1=n=0 \pmod 2 $, assurdo
oppure n=a, ma allora $ (2^a-1)^{n-1}-\dots+1=1 $, cioè n=1, assurdo
oppure a=0, ma allora q=0, assurdo
Spero di non aver scritto altre eresie xD
Avatar utente
Reginald
Messaggi: 137
Iscritto il: 24 gen 2009, 15:52
Località: Trento

Messaggio da Reginald »

Nono, adesso funziona.. :wink:
Rispondi